Q18

User avatar
 
ManhattanPrepLSAT1
Thanks Received: 1909
Atticus Finch
Atticus Finch
 
Posts: 2851
Joined: October 07th, 2009
 
 
 

Q18

by ManhattanPrepLSAT1 Mon Apr 26, 2010 12:32 pm

Here's a setup to help approach this question.

PT11, S1, G3, Q18_AtlasLSAT.pdf


The rule regarding J and M is too strong to not utilize. Since J is selected if, and only if, M is selected then either J and M are both selected or neither J and M are selected, creating two hypotheticals when the chairperson is a homeowner.

I didn't explain each step, but I think if you start with the constraint regarding J and M and then follow the rules, you'll see each inference in the setup.

Let me know if this doesn't make this question much easier. Also, when I approach this game, I have 4 frames almost entirely filled out. I don't have to do any more writing once I have them established. I base them off two splits. The first split is that there are either 2 tenants and 3 homeowners, or 3 tenants and 2 homeowners. Then for each, either J and M are both selected or neither are selected. This creates 4 frames with every possible solution before I even start with the questions.
 
catie0128
Thanks Received: 0
Forum Guests
 
Posts: 11
Joined: May 17th, 2011
 
 
 

Re: Diagram

by catie0128 Thu May 19, 2011 2:59 pm

When I do Q18, i am able to get JMKRS as those that can be selected, with either R or S as the chairperson. Due to this, I concluded that G and Q are not a "must" to be selected. I got C as the solution. What am I missing here?
User avatar
 
ManhattanPrepLSAT1
Thanks Received: 1909
Atticus Finch
Atticus Finch
 
Posts: 2851
Joined: October 07th, 2009
 
 
 

Re: Q18

by ManhattanPrepLSAT1 Tue Jul 12, 2011 10:59 am

I've attached a set of frames as well that might help on this question... The conditional question limits us to just two of the frames and only answer choice (A) must be true in the two applicable frames. Each of the remaining answer choices could be false.
catie0128 Wrote:When I do Q18, i am able to get JMKRS as those that can be selected, with either R or S as the chairperson.

The problem here is that there are many more possible solutions than that, so while answer choice (C) could be true, it doesn't have to be. Try avoiding completing the frame with information that could be true, and limit yourself to only things that must be true!

Hope that helps, and let me know if you have further questions on this one!

Image
 
pahuber2
Thanks Received: 0
Forum Guests
 
Posts: 1
Joined: March 29th, 2012
 
 
 

Re: Q18

by pahuber2 Thu Mar 29, 2012 6:39 pm

That is right on! Once you break the tenants into two paths everything becomes clear. Thanks.